K
Khách

Hãy nhập câu hỏi của bạn vào đây, nếu là tài khoản VIP, bạn sẽ được ưu tiên trả lời.

11 tháng 12 2018

Trùng trùng tên Mai Khương Duy thì phải nhỉ!!!!!!!!!!!!!!!!!!

11 tháng 12 2018

khoong phải tôi hnes

11 tháng 12 2018

\(P=x^{2}+y^{2}+\frac{1}{(4-\frac{1}{x}-\frac{1}{y})^{2}}\geq x^{2}+1+\frac{1}{(3-\frac{1}{x})^{2}}=x^{2}+1+\frac{x^{2}}{(3x-1)^{2}}\) ( do \(y\geq 1)\)

\(x> \frac{1}{3}=>3x-1> 0 \)

Áp dụng bất đẳng thức Cô-si cho 2 số dương: 

\(x^{2}+\frac{x^{2}}{4(3x-1)^{2}}\geq 2\sqrt{x^{2}.\frac{x^{2}}{4(3x-1)^{2}}}=\frac{x^{2}}{3x-1}\)

Ta cm: \(\frac{x^{2}}{3x-1}\geq \frac{1}{2}<=>2x^{2}\geq 3x-1<=>(x-1)(2x-1)\geq 0\) đúng do \(\frac{1}{3}< x\leq \frac{1}{2}\)

\(1+\frac{3x^{2}}{4(3x-1)^{2}}=\frac{1}{4}+\frac{3}{4}(1+\frac{x^{2}}{(3x-1)^{2}})\geq \frac{1}{4}+\frac{3}{4}.2.\frac{x}{3x-1}\geq \frac{1}{4}+\frac{3}{4}.2=\frac{7}{4}\)

Do \(\frac{x}{3x-1}=\frac{1}{3}.\frac{3x}{3x-1}=\frac{1}{3}(1+\frac{1}{3x-1})\geq \frac{1}{3}(1+\frac{1}{\frac{3}{2}-1})=1\)

\(<=>y=1,x=\frac{1}{2}\)

Phù ~ THỞ PHÀO NHẸ NHÕM

11 tháng 12 2018

TRẢ LỜI:

21+3+2004=2028

22+8+2004=2034

HOK TỐT NHÉ

TK NHÉ

I. Nội qui tham gia "Giúp tôi giải toán"

1. Không đưa câu hỏi linh tinh lên diễn đàn, chỉ đưa các bài mà mình không giải được hoặc các câu hỏi hay lên diễn đàn;

2. Không trả lời linh tinh, không phù hợp với nội dung câu hỏi trên diễn đàn.

3. Không "Đúng" vào các câu trả lời linh tinh nhằm gian lận điểm hỏi đáp.

Các bạn vi phạm 3 điều trên sẽ bị giáo viên của Online Math trừ hết điểm hỏi đáp, có thể bị khóa tài khoản hoặc bị cấm vĩnh viễn không đăng nhập vào trang web.

11 tháng 12 2018

Sửa lại đề: \(M=\frac{1}{\left(x-1\right)\left(2-x\right)}+\frac{1}{\left(x-1\right)^2}+\frac{1}{\left(2-x\right)^2}\)

12 tháng 12 2018

\(M=\frac{1}{\left(x-1\right)\left(2-x\right)}+\frac{1}{\left(x-1\right)^2}+\frac{1}{\left(2-x\right)^2}\ge3\sqrt[3]{\frac{1}{\left(x-1\right)^3\left(2-x\right)^3}}=\frac{3}{\left(x-1\right)\left(2-x\right)}\)

\(=\frac{-3}{x^2-3x+2}=\frac{-3}{\left(x^2-3x+\frac{9}{4}\right)-\frac{1}{4}}=\frac{-3}{\left(x-\frac{3}{2}\right)^2-\frac{1}{4}}\ge\frac{-3}{-\frac{1}{4}}=12\)

Dấu "=" xảy ra \(\Leftrightarrow\)\(\hept{\begin{cases}\frac{1}{\left(x-1\right)^2}=\frac{1}{\left(x-1\right)\left(2-x\right)}=\frac{1}{\left(2-x\right)^2}\\\left(x-\frac{3}{2}\right)^2=0\end{cases}\Leftrightarrow x=\frac{3}{2}}\)

... 

11 tháng 12 2018

Thôi không cần giúp nha , mình biết làm rồi , các bạn xem đúng chưa :

                                    Giải:

\(S=a+\frac{1}{a}=\frac{8a}{9}+\left(\frac{a}{9}+\frac{1}{a}\right)\ge\frac{24}{9}+2\sqrt{\frac{a}{9}.\frac{1}{a}}=\frac{10}{3}\)

Nghĩ mãi 10 phút mới ra

11 tháng 12 2018

Vì \(a\ge3\Rightarrow\hept{\begin{cases}a>0\\\frac{1}{a}>0\end{cases}}\)

Áp dụng bất đẳng thứ Cô si cho 2 số nguyên dương a;1/a ta có:

\(a+\frac{1}{a}\ge2\sqrt{a.\frac{1}{a}}=2\)\(\Rightarrow S\ge2\)

\(S=2\Leftrightarrow a=\frac{1}{a}\Leftrightarrow a^2=1\Leftrightarrow a=1\)(vì a>0)

Vậy: \(minS=2\Leftrightarrow a=1\)

11 tháng 12 2018

Áp dụng BĐT AM-GM ta có:

\(\frac{a}{b}+\frac{b}{c}+\frac{c}{a}\ge3.\sqrt[3]{\frac{a}{b}.\frac{b}{c}.\frac{c}{a}}=3.\sqrt[3]{1}=3\)

                                                       đpcm